[size=100]As the title reveals, I have been preparing for LSAT for quite some time, but recently decided to apply for Business Schools and switched to GMAT.
Meanwhile I have gotten used to the rigor of LSAT Logical Reasoning questions. The last couple days I went through all GMAT Critical Reasoning questions from Kaplan's GMAT 800, Manhattanprep's AllGMAT Verbal book and Manhattanprep's Question Bank. For many of the questions I got wrong, I am still convinced that more than 1 of the answers could equally be treated as correct (even if the stem says "most reasonable", "most strengthens/weakens" etc.), and I am convinced that they would definitely not have been precise enough to be set for LSAT.
Therefore, I thought to initiate a thread where we can post GMAT CR questions that are too unprecise. I'll make the start:
Manhattanprep Verbal book:
Chapter 22, Pset1:
Premise 1: Museum A display --> undamaged + proven to be authentic
Premise 2: Doubts about Mycenaean vase's authenticity, which is currently on display
Premise 3: prove vase's authenticity --> pulverize + spectroscopic analysis
Conclusion: ?
In my opinion, both B and C are true!
B: If the museum were sufficiently diligent in proving the authenticity of the vase before displaying it, then it means they would have pulverized and sent it to spectroscopy (see premise 3). If they had done that, they would not have been able to display it (since it has suffered from damage). But they are displaying it (see premise 2), therefore they were definitely NOT sufficiently diligent.
C: Indeed, it will no longer display it, based on the same logic I wrote for answer B.
The only reason which comes to my mind of eliminating B is because the stimulus says "will display", indicating that the statements might be not valid for the moment, but for the future only. This is really too vague though, on LSAT it would have been very clearly indicated that it refers to future only if so.
Manhattanprep Question Bank:
Q19:
Premise: TV is becoming too expensive for typical consumer
Conclusion: state that TV/household > persons/household will be reversed
In my opinion, both A and C strengthen equally (or at least I can't see any difference why one strengthens more than the other)!
A: this trend could definitely be due to the fact that consumers begin not being able to afford a TV. Potential objections:
- a rise of 300% within one year is quite big, it might be due to another reason, or an outlier
- the rise might be quite high, but it can be the case that the percentage of the population conducting parties for those purposes is so small that even a 300% increase can't be strong enough to reverse the trend in the conclusion
C: Indeed, this can also be due to the expensiveness of TV. Potential objections:
- TV can also be used as a secondary source of information and entertainment
- how much is the increase in purchases of the other devices? 0.1%, 1%, 10%, 50%?
Neither answer is completely bullet-proof (they don't have to, as stem says "most" strongly supports), but I can't really say that one answer is more bullet-proof (= requires more assumptions) than the other.
Kaplan GMAT 800:
This book is generally not very rigorous. Quite many questions were vague here and would certainly not make it through LSAT. I'll only write those down where I definitely think more than 1 answers are correct.
Q7:
Frankly, I've never seen answer choices so vaguely related to the stimulus in the LSAT. If we think very rigorously about it, none of them (except maybe E) really needs to be assumed. But if we compare the choices with each other, let's compromise a bit in terms of rigor and focus on (B) and (C) only:
For me, if we agree that C does not have to be assumed, then this is also true for B! I agree with the explanation given for why C is not a necessary assumption, but we can apply the same logic to B: the argument is only talking about candidates who are committed in their political party! The assumption has to be that Adele Richardson is likely to support the policy decision made by the national leadership of her minor party, but not the candidate of ANY party! Maybe candidates of other parties do not support policies of their own party at all...
Q5 also deserves an honourable mention here, but rather because it is so weirdly and vaguely formulated that it is not easy to follow the logic intended by the writer of the argument.[/size=100]